Measure Theory/Section 1 Proofs, Measure

From Wikiversity
Jump to navigation Jump to search

Section 1 Proofs, Measure[edit | edit source]

Proofs are organized into subproofs. If any statement is followed by a boxed region, then the boxed region is a subproof of the statement.

Lesson 1: No Total Measure of Real Numbers[edit | edit source]

Theorem: There Is No Total Measure of Real Numbers

Let  be any real-valued set function.  Then  must not satisfy at least one of the properties: Length measure, nonnegativity, translation-invariance, countable additivity.

Proof:

Assume that there is a function which has the properties of nonnegativity, interval length, translation-invariant, and countably additive.

This implies a contradiction.

Define the relation on by if .

is reflexive.

Let . Then and therefore by definition.

is symmetric.

Let so and by definition.

Then by the closure of under multiplication.

Then by definition.

is transitive.

Let and so that by definition and all three are in (0,1).

Then by the closure of under addition.

Then by definition .

Because is reflexive, symmetric, and transitive, therefore it is an equivalence relation.

Because is an equivalence relation there is a partition P induced by . If then write the cell of P containing x by .

Define F to be any arbitrary set with the following property. if and only if there is a unique such that .

Let be any enumeration of , which must exist because the set is countable.

For any two distinct , the sets and are disjoint.

Suppose there is some .

Then .

Let and .

Then by the closure of rational numbers under subtraction.

Then and therefore .

Because F contains unique representatives of each cell of the partition P, then .

Returning to an earlier equation, this implies

Then and therefore since is an enumeration, .

Define .

Then .

Let .

Let such that and therefore .

Also since both .

Therefore there is some in the enumeration , such that .

Then .

Let , and therefore there is some in the enumeration , and then there is some such that .

Since both and then .

is monotonic because it is countably additive and monotonic.

Let .

Then .

By countable additivity .

By the nonnegativity of ,

By monotonicity and the interval length property,

By countable additivity and the disjointness of ,

By translation-invariance, there is some constant such that every .

By all of the above,

If then , and if then from the above.

From all of the above, we have established that

  • or .

This is a contradiction.

Lesson 2: Outer Measure Properties[edit | edit source]

Theorem: Outer-measure Is Well-defined and Nonnegative.

For every  the outer-measure takes a unique extended real number value.  That is to say .  Also this value is nonnegative, .  

Proof:

always exists as an extended real number, and .

is an open set and .

Therefore is an open interval over-approximation of A.

Therefore is an over-estimate of A.

Therefore the set of over-estimates of A is non-empty.

Also the set of over-estimates of A is bounded below by zero.

If is any open interval over-approximation of A, and the corresponding over-estimate, then e is a sum of nonnegative numbers.

Therefore .

Therefore the infimum over the set of over-estimates of A exists and is at least zero.

Theorem: Outer Measure Is Determined by Countable Sums

For any , 

  

Proof:

For any uncountable series of nonnegative numbers, either the sum is infinity or at most countably many terms are nonzero. The proof of this fact is deferred to Terence Tao's book Analysis II.

Let be an uncountable over-approximation of A.

Either or there are only countably many nonzero terms, .

Because the length of an open interval is always strictly positive, then .

But also, is a countable series and is an over-estimate of A.

Therefore if is the collection of all over-estimates of A, and if is the set of over-estimates which are countable series, then .

Therefore .

Theorem: Outer-measure Is Translation Invariant.

For every subset  and real number  the outer-measure of A is invariant under translation by c, 
  

Proof:

Let be a set of real numbers, and any real number.

Let be any open interval over-approximation of A.

Define

Then is an open interval over-approximation of .

If then is an open interval. Hence all the elements of are open intervals.

If then there is some such that .

Then there is some such that .

Then and .

So covers A.

Let be the set of all over-estimates of A.

Likewise define to be the set of all over-estimates of .

For every over-estimate of A, and for every , we have that .

Therefore .

Therefore and so .

Mutatis mutandis the same proof shows .

Therefore and therefore

Theorem: Outer-measure Is Monotonic.

If  then .

Proof:

Let .

Any open interval over-approximation of B is immediately also an open interval over-approximation of A, which follows trivially by definitions.

Set as the set of over-estimates of A and the over-estimates of B.

Then .

From elementary analysis, therefore

Theorem: Countable Sets Are Null.

If  is a countable set, then .

If then we can set the open interval over-approximation .

Then

.

Therefore, for the rest of the proof, assume .

Let be a countable set, with enumeration .

Let .

Then .

Define the sequence of open intervals by

...
...

If N is non-finite with final indexed element then for indices set .

Then .

Then as a geometric sum

From an elementary argument it is clear that is an open interval over-approximation of N.

Therefore by definition as an infimum, .

Therefore .

Lesson 3: Outer Measure Interval Length[edit | edit source]

Theorem: Outer Measure Interval Length.

For every interval , its outer-measure is its length, .

Proof:

Let be any interval with end-points a and b. Note that the bounds are extended real numbers, so a may be and b may be .

Note that if then I is a countable set, and by a previous result, has measure zero which is equal to its length. Therefore, throughout the rest of the proof, we assume .

If is any closed, bounded interval of real numbers, then .

Let be a positive real number less than .

Then .

Define the open interval over-approximation .

Then

Letting we have .

Also is a lower bound on the set of all over-estimates of I.

Let be any over-estimate of I.

Let be a finite subcover of I, which must exist because [a,b] is a compact set and an open cover.

Let be the subset which results from successively removing from any interval which is a subset of some other interval.

is still a cover of I.

List the elements in increasing order of the left end-point. for .

For any , if then either or .

Either case is impossible because no interval can be a subset of any other, hence we must have the strict inequality .

Because a is covered by the set, then it is in one of the intervals. Whichever one it is, we must then have .

Mutatis mutandis, the same argument shows .

For each , if then is not covered by any interval in . Therefore .

Then by merely shifting parentheses for regrouping terms,

where the inequalities indicated by curly braces are due to . The inequalities and are due to earlier inequalities.

Also

because each series is of nonnegative terms, and contains a subsequence of the terms in the series before it.

Combining all of the above,

By definition of the outer measure as an infimum, then

Therefore

If is a bounded open interval then .

By monotonicity .

Let be any positive real number less than .

Then by monotonicity,

Letting we have

Up to this point we now have shown that every bounded open interval, and every bounded closed interval, has outer measure equal to its length.

If I is any other bounded interval, then

so that by monotonicity

and so .

If I is any interval unbounded on the right but finite on the left, then for every we have .

By monotonicity

as

Mutatis mutandis the same argument shows that if I is finite on the right and unbounded on the left, or if , then .

In every case, therefore, .

Lesson 4: Outer Measure Subadditivity[edit | edit source]

Theorem: Outer-measure not Additive

 is not countably additive.

Proof:

In earlier theorems we have proved that satisfies the properties Nonnegativity, Interval length, and Translation invariance.

We also know from an earlier theorem that no function, defined on , has all four properties, Nonnegativity, Interval length, Translation invariance, and Countable additivity.

Since is defined on then it cannot be countably additive.

Theorem: Outer-measure Subadditivity.

Let  be any countable sequence of subsets of real numbers.  Then 
  

Proof:

Suppose that for some we have .

The sum of with any nonnegative number is again .

Therefore, regardless of the value of the left-hand side, we have

Therefore throughout the rest of the proof, assume that each has finite outer measure.

Let be any positive real number.

For each there is an open interval over-approximation of , call it , such that

Then is an open interval over-approximation of .

Also,

Therefore by definition of the infimum,

Letting

Theorem: Null Adding and Subtracting.

Let  be two subsets and E a null set.  Then 
  

Proof:

By monotonicity, and the fact that ,

Therefore .

From this result,

Lesson 5: Measurable Sets[edit | edit source]

Theorem: Null Sets Are Measurable.

Every null set is measurable.

Proof:

Let be a null set.

Let be any set.

By monotonicity therefore

By a previous result, because E is null, .

Therefore

so E splits A cleanly.

Since A was arbitrary, therefore E is measurable.

Theorem: Measurable Sets Closed Under Complement.

If  is a measurable subset then  is measurable.

Proof:

Let be a measurable set.

Let be any set of real numbers.

Then by definition of the set difference,

So splits A cleanly, and since A was arbitrary, then is measurable.

Theorem: Open Rays Are Measurable.

For every  the open ray  is measurable.

Proof:

Let , and let be any set of real numbers. Let be the open ray to the right of a.

If then I splits A cleanly.

By monotonicity, .

If then regardless of the right-hand side we must have .

Therefore for the rest of this proof, assume is finite.

Let be any positive real number.

Then .

By definition as a finite infimum, there is an open interval over-approximation of A such that

Define and .

Then and are open interval over-approximations of and respectively.

Also

Also .

Let where the end-points are real numbers.

If then and and then and .

Then

Mutatis mutandis the same proof handles the case where .

If then and . Then and .

So we have shown in all cases that .

Therefore

Letting

Therefore .

Therefore I splits A cleanly.

If then I splits A cleanly.

Because is countable, therefore it is a null set, due to a theorem from an earlier lesson.

Therefore we may union and subtract from a set without changing its outer measure.

Denote .

Therefore, by all of the above,

Since I splits A cleanly in all cases, and A was arbitrary, then I is measurable.

Lesson 6: Measurable Sets Are a Sigma-algebra[edit | edit source]

Theorem: The Measurable Sets Form a -algebra.

The collection  is a -algebra.

Proof:

From two earlier theorems, because countable sets are null and null sets are measurable.

Also from an earlier theorem, is closed under taking complements.

If are two measurable sets then their union is measurable, .

Let

Let be any set of real numbers.

Then splits A cleanly.

By the measurability of E followed by the measurability of F, followed by elementary set theory,

By monotonicity and elementary set theory,

With the further observation by elementary set theory that and , then all of the above now shows

Therefore .

If is any finite sequence of measurable sets then their union is measurable,

If the case is trivial.

Suppose the theorem is true for some , and consider the case for sets.

Then .

Then by the result which we proved above for two sets,

Thus the proof by induction is complete.

If is any countable sequence of measurable sets then .

Let .

Let .

Also let and for define so that forms a sequence of disjoint sets.

Each is measurable by the result that complements and finite unions of measurable sets are measurable.

Also .

By subadditivity,

For each

Moreover, .

Lemma: Finite Split by Measurables

If are any finite sequence of disjoint measurable sets and is any set of real numbers, then

Proof:

If are any two disjoint measurable sets and is any set of real numbers, then .

Because H is measurable then

If is any finite sequence of disjoint measurable sets, and is any set of real numbers, then

The case for is trivial.

Suppose the claim holds for some and now consider the claim for sets.

Therefore

Now taking the limit on each side of this last inequality,

By subadditivity,

Putting together results from above,

Since A was arbitrary, this shows that E splits every set cleanly, and therefore E is measurable.

The above shows that

  • .
  • is closed under complement and countable unions.

Therefore is a -algebra.

Theorem: -algebra Space and Intersection

Let X be any set and  a -algebra on X.  Then  and  is closed under countable intersections.

Proof:

By closure under complements, .

By de Morgan's law, if then

By the closure under complements, each is measurable.

By closure under unions, is measurable.

By closure under complements, is measurable.

Theorem: Intervals Are Measurable.

If  is an interval then .

Proof:

From an earlier result, all open rays to the right, , are measurable.

Let be any closed ray to the left.

Then is the complement of an open ray to the right.

By the closure of measurable sets under complements, therefore I is measurable.

Now let a bounded interval open on the left, for two real numbers.

Then , and by closure of measurable sets under intersections, then J is measurable.

Now let be any bounded open interval.

Then define the sequence of intervals open on the left, for .

Then and each is measurable.

Since the measurable sets are closed under countable unions, then K is measurable.

Mutatis mutandis the same basic proof demonstrates the measurability of all other remaining kinds of intervals.

Lesson 8: Properties of Length Measure[edit | edit source]

Theorem: Length-measure Is Countably Additive.

Let  be a disjoint sequence of measurable sets.  Then 
  

Proof:

By subadditivity,

Fix any .

In the following, the first inequality is by monotonicity. The second is due to the lemma from the previous section, Finite Split by Measurables, setting B in that lemma to .

Taking in the above shows .

Therefore .

Theorem: Length-measure Excision.

Let  be two measurable sets, and , and assume that .  Then .

Proof:

By monotonicity and hence is a finite real number. Therefore arithmetic operations are well-defined for and .

By additivity,

Whence

Theorem: Upward Continuity of Measure.

Let  be an ascending sequence of measurable sets.  Then 

  

Proof:

Define and for , then .

Then is a sequence of disjoint measurable sets, and .

Therefore by additivity,

Theorem: Downward Continuity of Measure.

Let  be a descending sequence of measurable sets.  If  is finite then 
  

Proof:

By monotonicity, every and therefore arithmetic operations on these are well-defined.

Define for each .

Then is an increasing sequence of measurable sets, and hence the upward continuity of measure applies to it.

By excision,

and

By upward continuity of measure, therefore the two objects above are equal and therefore .

Whence